4
$\begingroup$

I am working with the below functional

$S[y]=\alpha y(1)^2+ \int_0^1dx \beta y'^2, \:\:y'(0)=0$

with a natural boundary condition at $x=1$ and the constraint

$C[y] =\gamma y(1)^2 + \int_0^1dx w(x)y^2=1$,

where $\alpha$, $\beta$ and $\gamma$ are constants. The task is to show that the stationary paths satisfy an Euler-Lagrange equation

$\beta \frac{d^2y}{dx^2} + \lambda w(x) y=0, \:\:y(0)=0,(\alpha-\gamma \lambda)y(1) + \beta y'(1)=0$,

where $\lambda$ is a Lagrange multiplier. I then have to let $w(x)=1$ and $\alpha=\beta=\gamma=1$ and find the non-trivial stationary paths of this system, along with the eigenfunctions and the values for the associated Lagrange multiplier. Is anyone able to guide me through the process on this one?

$\endgroup$

1 Answer 1

1
$\begingroup$

The auxiliary functional is: $$\bar{S}[y] =(\alpha -\lambda\gamma)y(1)^2+\int_0^1(\beta y'^2-\lambda\omega(x)y^2 dx$$ Now determine: $\bar{S}[y+\epsilon h]$, with $y(0)=0$ Then the Gateaux differential becomes: $$\triangle \bar{S}[y,h] = 2(\alpha -\lambda\gamma)y(1) h(1) +2 \int_0^1(\beta y'h'-\lambda\omega(x)y)h dx$$ Now integration by parts gives:$$\triangle \bar{S}[y,h] = 2(\alpha -\lambda\gamma)y(1) h(1) + 2\beta y'(1)h(1)-2\int_0^1(\beta y''+\lambda\omega(x)y)h$$ Since $h(0)=0$. On a stationary path $\triangle \bar{S}[y,h]=0$ for all allowed h. Now for $h(1)=0$ as a subset use the fundamental lemma of the calculus of variations to obtain: $$\beta \frac{d^2y}{dx^2}+\lambda\omega(x)y=0$$ Then on a stationary path $2(\alpha -\lambda\gamma)y(1) + \beta y'(1)h(1)=0$ and this must be true for all h, including those for which $h(1) \neq 0$, hence we also need: $$(\alpha -\lambda\gamma)y(1) + \beta y'(1)= 0$$ which is required in your question.

$\endgroup$
8
  • $\begingroup$ I didn't expand $S[y+\epsilon h]$ because it is quite standard. If you' ve any difficulties in it, let me know. $\endgroup$ Commented May 10, 2018 at 10:40
  • $\begingroup$ @Tom: my answer doesn't suite your wishes? Please let me know how to improve it. $\endgroup$ Commented May 10, 2018 at 17:33
  • $\begingroup$ I guess you submitted your TMA answer without rewarding this contribution $\endgroup$ Commented May 12, 2018 at 18:19
  • $\begingroup$ @ Joe Goldiamond: I've been asked to look at this question, I don't do the course though! can you briefly explain why the equations for euler lagrange don't work here,and why we use the gateaux differential? $\endgroup$
    – jiboom
    Commented May 18, 2018 at 11:39
  • 1
    $\begingroup$ The EL are not suitable overhere due to the first part of the functional. I.e. $(\alpha -\lambda \gamma) y(1)^2$. You can’t apply the EL eq. $\endgroup$ Commented May 18, 2018 at 17:44

You must log in to answer this question.

Not the answer you're looking for? Browse other questions tagged .